$A$ and $B$ nonempty subsets of $mathbb{R}$, if $sup(A)=sup(B)$ then $forall a in A, exists b in B$ such that...












3












$begingroup$



Let $A$ and $B$ nonempty subsets of $mathbb{R}$, such that $sup(A)=sup(B)$ and $sup(A) not in A$



Prove: $forall a in A, exists b in B$ such that $a<b$




I started out with the fact that:
$$ a< sup (A) = sup (B)$$
And of course we know that
$$b leq sup(B)= sup(A)$$
So now we know that $exists b$ that sort of "must be in the middle"
$$ a <b leq sup(B) $$
I don't know how to make this concrete, also is this element simply the supremum?










share|cite|improve this question









$endgroup$








  • 1




    $begingroup$
    What is your definition of the supremum, and what properties have you proven for it? For example, if your definition of $mathrm{sup}(X)$ is that it is a number $s$ such that (i) $s$ is an upper bound for $X$; and (ii) if $t$ is any upper bound for $x$, then $sleq t$; then from $altmathrm{sup}(B)$ you can conclude that $a$ is not an upper bound for $B$, and therefore...
    $endgroup$
    – Arturo Magidin
    Dec 2 '18 at 0:00
















3












$begingroup$



Let $A$ and $B$ nonempty subsets of $mathbb{R}$, such that $sup(A)=sup(B)$ and $sup(A) not in A$



Prove: $forall a in A, exists b in B$ such that $a<b$




I started out with the fact that:
$$ a< sup (A) = sup (B)$$
And of course we know that
$$b leq sup(B)= sup(A)$$
So now we know that $exists b$ that sort of "must be in the middle"
$$ a <b leq sup(B) $$
I don't know how to make this concrete, also is this element simply the supremum?










share|cite|improve this question









$endgroup$








  • 1




    $begingroup$
    What is your definition of the supremum, and what properties have you proven for it? For example, if your definition of $mathrm{sup}(X)$ is that it is a number $s$ such that (i) $s$ is an upper bound for $X$; and (ii) if $t$ is any upper bound for $x$, then $sleq t$; then from $altmathrm{sup}(B)$ you can conclude that $a$ is not an upper bound for $B$, and therefore...
    $endgroup$
    – Arturo Magidin
    Dec 2 '18 at 0:00














3












3








3





$begingroup$



Let $A$ and $B$ nonempty subsets of $mathbb{R}$, such that $sup(A)=sup(B)$ and $sup(A) not in A$



Prove: $forall a in A, exists b in B$ such that $a<b$




I started out with the fact that:
$$ a< sup (A) = sup (B)$$
And of course we know that
$$b leq sup(B)= sup(A)$$
So now we know that $exists b$ that sort of "must be in the middle"
$$ a <b leq sup(B) $$
I don't know how to make this concrete, also is this element simply the supremum?










share|cite|improve this question









$endgroup$





Let $A$ and $B$ nonempty subsets of $mathbb{R}$, such that $sup(A)=sup(B)$ and $sup(A) not in A$



Prove: $forall a in A, exists b in B$ such that $a<b$




I started out with the fact that:
$$ a< sup (A) = sup (B)$$
And of course we know that
$$b leq sup(B)= sup(A)$$
So now we know that $exists b$ that sort of "must be in the middle"
$$ a <b leq sup(B) $$
I don't know how to make this concrete, also is this element simply the supremum?







real-analysis






share|cite|improve this question













share|cite|improve this question











share|cite|improve this question




share|cite|improve this question










asked Dec 1 '18 at 23:54









Wesley StrikWesley Strik

1,667423




1,667423








  • 1




    $begingroup$
    What is your definition of the supremum, and what properties have you proven for it? For example, if your definition of $mathrm{sup}(X)$ is that it is a number $s$ such that (i) $s$ is an upper bound for $X$; and (ii) if $t$ is any upper bound for $x$, then $sleq t$; then from $altmathrm{sup}(B)$ you can conclude that $a$ is not an upper bound for $B$, and therefore...
    $endgroup$
    – Arturo Magidin
    Dec 2 '18 at 0:00














  • 1




    $begingroup$
    What is your definition of the supremum, and what properties have you proven for it? For example, if your definition of $mathrm{sup}(X)$ is that it is a number $s$ such that (i) $s$ is an upper bound for $X$; and (ii) if $t$ is any upper bound for $x$, then $sleq t$; then from $altmathrm{sup}(B)$ you can conclude that $a$ is not an upper bound for $B$, and therefore...
    $endgroup$
    – Arturo Magidin
    Dec 2 '18 at 0:00








1




1




$begingroup$
What is your definition of the supremum, and what properties have you proven for it? For example, if your definition of $mathrm{sup}(X)$ is that it is a number $s$ such that (i) $s$ is an upper bound for $X$; and (ii) if $t$ is any upper bound for $x$, then $sleq t$; then from $altmathrm{sup}(B)$ you can conclude that $a$ is not an upper bound for $B$, and therefore...
$endgroup$
– Arturo Magidin
Dec 2 '18 at 0:00




$begingroup$
What is your definition of the supremum, and what properties have you proven for it? For example, if your definition of $mathrm{sup}(X)$ is that it is a number $s$ such that (i) $s$ is an upper bound for $X$; and (ii) if $t$ is any upper bound for $x$, then $sleq t$; then from $altmathrm{sup}(B)$ you can conclude that $a$ is not an upper bound for $B$, and therefore...
$endgroup$
– Arturo Magidin
Dec 2 '18 at 0:00










4 Answers
4






active

oldest

votes


















2












$begingroup$

The following argument utilizes the second characterization of the supremum found here:



Suppose $a in A$. Since $sup(A) notin A$, we know that $sup(A)-a>0$. So we select $varepsilon=sup(A)-a$. Now we have that there exists $b in B$ such that
begin{aligned} b&>sup(B)-varepsilon \& = sup(B)-left(sup(A)-aright) \& = sup(B)-sup(B)+a \& =a.
end{aligned}






share|cite|improve this answer









$endgroup$









  • 1




    $begingroup$
    This is a good tool, yes we had to prove this as an earlier exercise in the book. It's a little bit less used than " the minimum of all upper bounds" property, but very useful indeed. Thank you.
    $endgroup$
    – Wesley Strik
    Dec 2 '18 at 10:27



















3












$begingroup$

By contradiction. Assume $exists a in A$ such that $forall b in B$, $bleq a$. Then $sup(B) leq a < sup(A)$, which is a contradiction.






share|cite|improve this answer









$endgroup$





















    3












    $begingroup$

    We know that $ ale sup A=sup B$, and because $sup A notin A$, $a< sup A=sup B$. In particular, because $sup B$ is the least upper bound of $B$, and $a$ is strictly less than it, there exists $bin B$ with $b>a$. If this were not the case, then $a$ would be a smaller lower bound for $B$, which is contradictory.






    share|cite|improve this answer









    $endgroup$













    • $begingroup$
      oh. I get it now!
      $endgroup$
      – Wesley Strik
      Dec 2 '18 at 13:32



















    2












    $begingroup$

    Note that $a<sup(A)=sup(B)$, so $a$ is not an upper bound of $B$, because $sup(B)$ is the least upper bound of $B$. Therefore $a<b$, for some $bin B$.






    share|cite|improve this answer









    $endgroup$













      Your Answer





      StackExchange.ifUsing("editor", function () {
      return StackExchange.using("mathjaxEditing", function () {
      StackExchange.MarkdownEditor.creationCallbacks.add(function (editor, postfix) {
      StackExchange.mathjaxEditing.prepareWmdForMathJax(editor, postfix, [["$", "$"], ["\\(","\\)"]]);
      });
      });
      }, "mathjax-editing");

      StackExchange.ready(function() {
      var channelOptions = {
      tags: "".split(" "),
      id: "69"
      };
      initTagRenderer("".split(" "), "".split(" "), channelOptions);

      StackExchange.using("externalEditor", function() {
      // Have to fire editor after snippets, if snippets enabled
      if (StackExchange.settings.snippets.snippetsEnabled) {
      StackExchange.using("snippets", function() {
      createEditor();
      });
      }
      else {
      createEditor();
      }
      });

      function createEditor() {
      StackExchange.prepareEditor({
      heartbeatType: 'answer',
      autoActivateHeartbeat: false,
      convertImagesToLinks: true,
      noModals: true,
      showLowRepImageUploadWarning: true,
      reputationToPostImages: 10,
      bindNavPrevention: true,
      postfix: "",
      imageUploader: {
      brandingHtml: "Powered by u003ca class="icon-imgur-white" href="https://imgur.com/"u003eu003c/au003e",
      contentPolicyHtml: "User contributions licensed under u003ca href="https://creativecommons.org/licenses/by-sa/3.0/"u003ecc by-sa 3.0 with attribution requiredu003c/au003e u003ca href="https://stackoverflow.com/legal/content-policy"u003e(content policy)u003c/au003e",
      allowUrls: true
      },
      noCode: true, onDemand: true,
      discardSelector: ".discard-answer"
      ,immediatelyShowMarkdownHelp:true
      });


      }
      });














      draft saved

      draft discarded


















      StackExchange.ready(
      function () {
      StackExchange.openid.initPostLogin('.new-post-login', 'https%3a%2f%2fmath.stackexchange.com%2fquestions%2f3022021%2fa-and-b-nonempty-subsets-of-mathbbr-if-supa-supb-then-forall%23new-answer', 'question_page');
      }
      );

      Post as a guest















      Required, but never shown

























      4 Answers
      4






      active

      oldest

      votes








      4 Answers
      4






      active

      oldest

      votes









      active

      oldest

      votes






      active

      oldest

      votes









      2












      $begingroup$

      The following argument utilizes the second characterization of the supremum found here:



      Suppose $a in A$. Since $sup(A) notin A$, we know that $sup(A)-a>0$. So we select $varepsilon=sup(A)-a$. Now we have that there exists $b in B$ such that
      begin{aligned} b&>sup(B)-varepsilon \& = sup(B)-left(sup(A)-aright) \& = sup(B)-sup(B)+a \& =a.
      end{aligned}






      share|cite|improve this answer









      $endgroup$









      • 1




        $begingroup$
        This is a good tool, yes we had to prove this as an earlier exercise in the book. It's a little bit less used than " the minimum of all upper bounds" property, but very useful indeed. Thank you.
        $endgroup$
        – Wesley Strik
        Dec 2 '18 at 10:27
















      2












      $begingroup$

      The following argument utilizes the second characterization of the supremum found here:



      Suppose $a in A$. Since $sup(A) notin A$, we know that $sup(A)-a>0$. So we select $varepsilon=sup(A)-a$. Now we have that there exists $b in B$ such that
      begin{aligned} b&>sup(B)-varepsilon \& = sup(B)-left(sup(A)-aright) \& = sup(B)-sup(B)+a \& =a.
      end{aligned}






      share|cite|improve this answer









      $endgroup$









      • 1




        $begingroup$
        This is a good tool, yes we had to prove this as an earlier exercise in the book. It's a little bit less used than " the minimum of all upper bounds" property, but very useful indeed. Thank you.
        $endgroup$
        – Wesley Strik
        Dec 2 '18 at 10:27














      2












      2








      2





      $begingroup$

      The following argument utilizes the second characterization of the supremum found here:



      Suppose $a in A$. Since $sup(A) notin A$, we know that $sup(A)-a>0$. So we select $varepsilon=sup(A)-a$. Now we have that there exists $b in B$ such that
      begin{aligned} b&>sup(B)-varepsilon \& = sup(B)-left(sup(A)-aright) \& = sup(B)-sup(B)+a \& =a.
      end{aligned}






      share|cite|improve this answer









      $endgroup$



      The following argument utilizes the second characterization of the supremum found here:



      Suppose $a in A$. Since $sup(A) notin A$, we know that $sup(A)-a>0$. So we select $varepsilon=sup(A)-a$. Now we have that there exists $b in B$ such that
      begin{aligned} b&>sup(B)-varepsilon \& = sup(B)-left(sup(A)-aright) \& = sup(B)-sup(B)+a \& =a.
      end{aligned}







      share|cite|improve this answer












      share|cite|improve this answer



      share|cite|improve this answer










      answered Dec 2 '18 at 0:22









      Matt A PeltoMatt A Pelto

      2,502620




      2,502620








      • 1




        $begingroup$
        This is a good tool, yes we had to prove this as an earlier exercise in the book. It's a little bit less used than " the minimum of all upper bounds" property, but very useful indeed. Thank you.
        $endgroup$
        – Wesley Strik
        Dec 2 '18 at 10:27














      • 1




        $begingroup$
        This is a good tool, yes we had to prove this as an earlier exercise in the book. It's a little bit less used than " the minimum of all upper bounds" property, but very useful indeed. Thank you.
        $endgroup$
        – Wesley Strik
        Dec 2 '18 at 10:27








      1




      1




      $begingroup$
      This is a good tool, yes we had to prove this as an earlier exercise in the book. It's a little bit less used than " the minimum of all upper bounds" property, but very useful indeed. Thank you.
      $endgroup$
      – Wesley Strik
      Dec 2 '18 at 10:27




      $begingroup$
      This is a good tool, yes we had to prove this as an earlier exercise in the book. It's a little bit less used than " the minimum of all upper bounds" property, but very useful indeed. Thank you.
      $endgroup$
      – Wesley Strik
      Dec 2 '18 at 10:27











      3












      $begingroup$

      By contradiction. Assume $exists a in A$ such that $forall b in B$, $bleq a$. Then $sup(B) leq a < sup(A)$, which is a contradiction.






      share|cite|improve this answer









      $endgroup$


















        3












        $begingroup$

        By contradiction. Assume $exists a in A$ such that $forall b in B$, $bleq a$. Then $sup(B) leq a < sup(A)$, which is a contradiction.






        share|cite|improve this answer









        $endgroup$
















          3












          3








          3





          $begingroup$

          By contradiction. Assume $exists a in A$ such that $forall b in B$, $bleq a$. Then $sup(B) leq a < sup(A)$, which is a contradiction.






          share|cite|improve this answer









          $endgroup$



          By contradiction. Assume $exists a in A$ such that $forall b in B$, $bleq a$. Then $sup(B) leq a < sup(A)$, which is a contradiction.







          share|cite|improve this answer












          share|cite|improve this answer



          share|cite|improve this answer










          answered Dec 2 '18 at 0:04









          mlerma54mlerma54

          1,177148




          1,177148























              3












              $begingroup$

              We know that $ ale sup A=sup B$, and because $sup A notin A$, $a< sup A=sup B$. In particular, because $sup B$ is the least upper bound of $B$, and $a$ is strictly less than it, there exists $bin B$ with $b>a$. If this were not the case, then $a$ would be a smaller lower bound for $B$, which is contradictory.






              share|cite|improve this answer









              $endgroup$













              • $begingroup$
                oh. I get it now!
                $endgroup$
                – Wesley Strik
                Dec 2 '18 at 13:32
















              3












              $begingroup$

              We know that $ ale sup A=sup B$, and because $sup A notin A$, $a< sup A=sup B$. In particular, because $sup B$ is the least upper bound of $B$, and $a$ is strictly less than it, there exists $bin B$ with $b>a$. If this were not the case, then $a$ would be a smaller lower bound for $B$, which is contradictory.






              share|cite|improve this answer









              $endgroup$













              • $begingroup$
                oh. I get it now!
                $endgroup$
                – Wesley Strik
                Dec 2 '18 at 13:32














              3












              3








              3





              $begingroup$

              We know that $ ale sup A=sup B$, and because $sup A notin A$, $a< sup A=sup B$. In particular, because $sup B$ is the least upper bound of $B$, and $a$ is strictly less than it, there exists $bin B$ with $b>a$. If this were not the case, then $a$ would be a smaller lower bound for $B$, which is contradictory.






              share|cite|improve this answer









              $endgroup$



              We know that $ ale sup A=sup B$, and because $sup A notin A$, $a< sup A=sup B$. In particular, because $sup B$ is the least upper bound of $B$, and $a$ is strictly less than it, there exists $bin B$ with $b>a$. If this were not the case, then $a$ would be a smaller lower bound for $B$, which is contradictory.







              share|cite|improve this answer












              share|cite|improve this answer



              share|cite|improve this answer










              answered Dec 2 '18 at 0:05









              Antonios-Alexandros RobotisAntonios-Alexandros Robotis

              9,90241640




              9,90241640












              • $begingroup$
                oh. I get it now!
                $endgroup$
                – Wesley Strik
                Dec 2 '18 at 13:32


















              • $begingroup$
                oh. I get it now!
                $endgroup$
                – Wesley Strik
                Dec 2 '18 at 13:32
















              $begingroup$
              oh. I get it now!
              $endgroup$
              – Wesley Strik
              Dec 2 '18 at 13:32




              $begingroup$
              oh. I get it now!
              $endgroup$
              – Wesley Strik
              Dec 2 '18 at 13:32











              2












              $begingroup$

              Note that $a<sup(A)=sup(B)$, so $a$ is not an upper bound of $B$, because $sup(B)$ is the least upper bound of $B$. Therefore $a<b$, for some $bin B$.






              share|cite|improve this answer









              $endgroup$


















                2












                $begingroup$

                Note that $a<sup(A)=sup(B)$, so $a$ is not an upper bound of $B$, because $sup(B)$ is the least upper bound of $B$. Therefore $a<b$, for some $bin B$.






                share|cite|improve this answer









                $endgroup$
















                  2












                  2








                  2





                  $begingroup$

                  Note that $a<sup(A)=sup(B)$, so $a$ is not an upper bound of $B$, because $sup(B)$ is the least upper bound of $B$. Therefore $a<b$, for some $bin B$.






                  share|cite|improve this answer









                  $endgroup$



                  Note that $a<sup(A)=sup(B)$, so $a$ is not an upper bound of $B$, because $sup(B)$ is the least upper bound of $B$. Therefore $a<b$, for some $bin B$.







                  share|cite|improve this answer












                  share|cite|improve this answer



                  share|cite|improve this answer










                  answered Dec 2 '18 at 0:30









                  egregegreg

                  180k1485202




                  180k1485202






























                      draft saved

                      draft discarded




















































                      Thanks for contributing an answer to Mathematics Stack Exchange!


                      • Please be sure to answer the question. Provide details and share your research!

                      But avoid



                      • Asking for help, clarification, or responding to other answers.

                      • Making statements based on opinion; back them up with references or personal experience.


                      Use MathJax to format equations. MathJax reference.


                      To learn more, see our tips on writing great answers.




                      draft saved


                      draft discarded














                      StackExchange.ready(
                      function () {
                      StackExchange.openid.initPostLogin('.new-post-login', 'https%3a%2f%2fmath.stackexchange.com%2fquestions%2f3022021%2fa-and-b-nonempty-subsets-of-mathbbr-if-supa-supb-then-forall%23new-answer', 'question_page');
                      }
                      );

                      Post as a guest















                      Required, but never shown





















































                      Required, but never shown














                      Required, but never shown












                      Required, but never shown







                      Required, but never shown

































                      Required, but never shown














                      Required, but never shown












                      Required, but never shown







                      Required, but never shown







                      Popular posts from this blog

                      Plaza Victoria

                      In PowerPoint, is there a keyboard shortcut for bulleted / numbered list?

                      How to put 3 figures in Latex with 2 figures side by side and 1 below these side by side images but in...